Derivación de las ecuaciones de Maxwell a partir del lagrangiano del tensor de campo

Empecé a leer Peskin y Schroeder en mi tiempo libre, y estoy un poco confundido acerca de cómo obtener las ecuaciones de Maxwell a partir de la densidad lagrangiana (sin fuente). L = 1 4 F m v F m v (dónde F m v = m A v v A m es el tensor de campo).

Sustituyendo la definición del tensor de campo se obtiene L = 1 2 [ ( m A v ) ( m A v ) ( m A v ) ( v A m ) ] . Sé que debería estar usando A m como la variable dinámica en las ecuaciones de Euler-Lagrange, que se convierten en L A m m L ( m A v ) = m L ( m A v ) , pero estoy confundido acerca de cómo proceder desde aquí.

Sé que debería terminar con m F m v = 0 , pero no veo muy bien por qué. Ya que m y v son índices ficticios, debería poder cambiarlos: ¿cómo se relacionan los índices en el lagrangiano con los índices en las derivadas en las ecuaciones de Euler-Lagrange?

Ver en el libro de Sean carrol. Lleno de derivación allí
¿Por qué no es suficiente tapar el F m v = [ m A v ] en las ecuaciones de Maxwell y demostrar que se cumplen?

Respuestas (5)

Variamos la acción

d L d t = d Λ ( A v , m A v ) d 3 X d t = 0
Λ ( A v , m A v ) es la densidad de lagrangiana del sistema.

Asi que,

( Λ A v d A v + Λ ( m A v ) d ( m A v ) ) d 3 X d t = 0
Integrando por partes obtenemos:
( Λ A v m Λ ( m A v ) ) d A v d 3 X d t = 0 Λ A v m Λ ( m A v ) = 0
Tenemos que determinar la densidad del lagrangiano. Uno de los términos trata de la interacción de las cargas con el campo electromagnético, j m A m . El otro término es la densidad de energía del campo electromagnético: este término es la diferencia del campo magnético y el campo eléctrico. Entonces tenemos:
Λ = j m A m + 1 4 m 0 F m v F m v
Tenemos:
Λ A v = j v
asi que:
m Λ ( m A v ) = 1 4 m 0 m ( ( m A v ) F k λ F k λ ) = 1 4 m 0 m ( ( m A v ) ( ( k A λ λ A k ) ( k A λ λ A k ) ) ) = 1 4 m 0 m ( ( m A v ) ( k A λ k A λ k A λ λ A k λ A k k A λ + λ A k λ A k ) )
El tercero y el cuarto son los mismos términos del primero y del segundo. Tu puedes hacer k λ :
m Λ ( m A v ) = 1 2 m 0 m ( ( m A v ) ( k A λ k A λ k A λ λ A k ) ) .
Pero
( m A v ) ( k A λ k A λ ) = k A λ ( m A v ) ( k A λ ) + k A λ ( m A v ) ( k A λ ) = k A λ d k m d λ v + gramo k α gramo λ β k A λ ( m A v ) ( α A β ) = 2 m A v .

Tenemos:

( m A v ) ( k A λ λ A k ) = 2 v A m .

Asi que,

m ( Λ ( m A v ) ) = 1 m 0 m ( m A v v A m ) = 1 m 0 m F m v .
Las ecuaciones lagrangianas proporcionan las ecuaciones de Maxwell no homogéneas:

m F m v = m 0 j v .

FYI, esta respuesta (v4) usa implícitamente la convención de signos ( + , , , ) .

Bueno, ya casi estás allí. Usa el hecho de que

( m A v ) ( ρ A σ ) = d m ρ d v σ
lo cual es válido porque m A v son d 2 componentes independientes.

¿Cómo probarlo realmente?

Estimado amc, primero, escribe tu densidad lagrangiana como

L = 1 4 F m v F m v = 1 2 ( m A v ) F m v
¿Está bien hasta ahora? los F m v contiene dos términos que la hacen antisimétrica en los dos índices. Sin embargo, se multiplica por otro F m v eso ya es antisimétrico, así que no necesito volver a antisimetrizarlo. En cambio, ambos términos me dan lo mismo, por lo que el coeficiente 1 / 4 simplemente cambia a 1 / 2 .

Ahora, las ecuaciones de campo te obligan a calcular las derivadas del Lagrangiano con respecto a A m y sus derivados. En primer lugar, la derivada del Lagrangiano L con respecto a A m componentes mismos se desvanece porque el Lagrangiano solo depende de las derivadas parciales de A m . ¿Está claro hasta ahora?

Entonces las ecuaciones de movimiento serán

0 = m [ L / ( m A v ) ] =
Ups, ya llegaste a este punto. Pero ahora, mira mi forma del Lagrangiano arriba. La derivada del Lagrangiano con respecto a m A v es simple
1 2 F m v
porque m A v simplemente aparece como un factor, por lo que las ecuaciones de movimiento simplemente serán
0 = + 1 2 m F m v
Sin embargo, deliberadamente he cometido un error. Sólo he diferenciado el Lagrangiano con respecto a m A v incluido en el primer factor de F m v , con los índices más bajos. Sin embargo, m A v Los componentes también aparecen en F m v , el segundo factor en el Lagrangiano, uno con los índices superiores. Si agrega los términos correspondientes de la regla de Leibniz, el resultado es simplemente que la contribución total se duplicará. Entonces, la ecuación correcta de movimiento, incluido el coeficiente natural, será
0 = m F m v
La normalización general es importante porque esta ecuación puede obtener términos adicionales, como la corriente, cuyo coeficiente es obvio, y no desea obtener un error relativo de dos entre la derivada de F y la corriente j .

Saludos Lubos

Oye, sé que esto tiene 5 años de retraso, pero tal vez veas esto: ¿Por qué ( m ϕ ) ( m A v F m v ) = F m v . ¿El tensor no depende también de las derivadas parciales? Entonces, ¿no tenemos que usar la regla del producto?
Hola @ usuario17574: ¿"cuál" tensor no depende de las derivadas parciales? Seguramente el tensor tensión-energía lo hace, y también el lagrangiano. Por eso su derivada con respecto a las derivadas parciales es distinta de cero. La derivada se calcula en la respuesta. La regla del producto sí funciona y es por eso que uno cancela el factor de 1 / 2 . ¿Has intentado leer la respuesta?
Sé que esta es una vieja pregunta, pero tengo una duda. Después de aplicar la regla de Leibniz obtenemos:
L ( m A v ) = 1 2 ( m A v F m v ( m A v ) + m A v ( m A v ) F m v )
¿Cómo estoy destinado a diferenciar el F m v en el primer término de la ecuación? No entiendo cómo hacerlo me dará F m v .
Debe usar un nuevo par de índices en lugar de copiar mu-nu tres veces. Después F α β / ( γ A d ) = gramo α γ gramo β d α β simplemente porque F es solo la diferencia entre dos términos similares (antisimetrización) y cada término tiene la derivada que es básicamente un delta de Kronecker, pero aquí con los índices elevados (para convertirse en la métrica de índice superior) debido a la ubicación de los índices en la expresión original .

Aunque tarde en la fiesta, publico una respuesta en un nivel elemental. Puede ser que esto demuestre el poder del cálculo tensorial utilizado en todas las buenas respuestas anteriores.

Resumen

En esta respuesta, intentaremos derivar las ecuaciones de Maxwell en el espacio vacío.

(001a) × mi = B t (001b) × B = m 0 j + 1 C 2 mi t (001c) mi = ρ ϵ 0 (001d) B = 0
de las ecuaciones de Euler-Lagrange
(002) t ( L η ˙ ȷ ) + [ L ( η ȷ ) ] L η ȷ = 0 , ( ȷ = 1 , 2 , 3 , 4 )
dónde
(003) L = L ( η ȷ , η ˙ ȷ , η ȷ ) ( ȷ = 1 , 2 , 3 , 4 )
es la densidad lagrangiana de la pregunta (excepto un factor constante)
(004) L = mi 2 C 2 B 2 2 + 1 ϵ 0 ( ρ ϕ + j A )
y η ȷ ( X 1 , X 2 , X 3 , t ) , ȷ = 1 , 2 , 3 , 4 los componentes A 1 , A 2 , A 3 , ϕ del potencial EM 4-vector respectivamente. En cierto sentido, esta derivación se basa en la inversa (: esto de encontrar una densidad lagrangiana adecuada a partir de las ecuaciones de Maxwell) moviéndose hacia atrás, vea mi respuesta aquí: Derivar la densidad lagrangiana para el campo electromagnético

1. Sección principal

Primero expresamos mi , B de (004) en términos de los componentes potenciales de 4 vectores A 1 , A 2 , A 3 , ϕ

(005a) B = × A (005b) mi = ϕ A t = ϕ A ˙
A partir de (005) las ecuaciones de Maxwell (001a) y (001d) son válidas automáticamente. Entonces, las cuatro (4) ecuaciones escalares de Maxwell (001b) y (001c) deben derivarse de las cuatro (4) ecuaciones escalares de Euler-Lagrange (002). Además, es razonable suponer que la ecuación vectorial (001b) debe derivarse de (002) con respecto a los componentes del potencial vectorial A = ( A 1 , A 2 , A 3 ) , mientras que la ecuación escalar (001c) debe derivarse de (002) con respecto al potencial escalar ϕ .

A partir de las ecuaciones (005) expresamos la densidad lagrangiana (004) en términos de los componentes potenciales de 4 vectores A 1 , A 2 , A 3 , ϕ :

(006a) mi 2 = ϕ A t 2 = A ˙ 2 + ϕ 2 + 2 ( ϕ A ˙ ) (006b) B 2 = × A 2 k = 1 k = 3 [ A k 2 A X k A k ]
La segunda ecuación en (006b), que es la identidad
(Id-01) × A 2 k = 1 k = 3 [ A k 2 A X k A k ]
se prueba en 2. Sección Identidades . Insertando expresiones (006) en (004) la densidad lagrangiana es
(007) L = 1 2 A ˙ 2 + 1 2 ϕ 2 + ϕ A ˙ 1 2 ϕ A t 2 1 2 C 2 k = 1 k = 3 [ A k 2 A X k A k ] × A 2 + 1 ϵ 0 ( ρ ϕ + j A )

Reorganizamos los elementos en (007) de la siguiente manera:

(008a) L = 1 2 ϕ 2 ρ ϕ ϵ 0 + ϕ A ˙ L ϕ = con respecto a  ϕ + 1 2 A ˙ 2 + 1 2 C 2 k = 1 k = 3 [ A X k A k A k 2 ] + j A ϵ 0 (008b) L = 1 2 ϕ 2 ρ ϕ ϵ 0 + ϕ A ˙ + 1 2 A ˙ 2 + 1 2 C 2 k = 1 k = 3 [ A X k A k A k 2 ] + j A ϵ 0 L A = con respecto a  A

los L ϕ parte de la densidad contiene todos ϕ -términos y participará razonablemente solo en la derivación de la ecuación de Maxwell (001c) a partir de la ecuación de Euler-Lagrange (002) con respecto a η 4 = ϕ . los L A parte de la densidad contiene todos A -términos y participará razonablemente solo en la derivación de la ecuación de Maxwell (001b) a partir de las ecuaciones de Euler-Lagrange (002) con respecto a η 1 , η 2 , η 3 = A 1 , A 1 , A 3 . Tenga en cuenta el término común ϕ A ˙ de las partes L ϕ , L A .

La ecuación de Euler-Lagrange con respecto a η 4 = ϕ es :

(009) t ( L ϕ ˙ ) 0 + [ L ( ϕ ) ] ϕ + A ˙ L ϕ ρ ϵ 0 = 0
o
(010) ( ϕ A t ) mi = ρ ϵ 0
esa es la ecuación de Maxwell (001c)
(001c) mi = ρ ϵ 0

Para derivar la ecuación de Maxwell (001b) la expresamos con la ayuda de las ecuaciones (005) en términos de los componentes de 4 vectores potenciales A 1 , A 2 , A 3 , ϕ :

(011) × ( × A ) = m 0 j + 1 C 2 t ( ϕ A t )
usando la identidad
(012) × ( × A ) = ( A ) 2 A
eq.(011) rendimientos
(013) 1 C 2 2 A t 2 2 A + ( A + 1 C 2 ϕ t ) = m 0 j
los k -componente de la ecuación (013) se expresa correctamente para parecerse a una ecuación de Euler-Lagrange de la siguiente manera:
(014) t ( A k t + ϕ X k ) + [ C 2 ( A X k A k ) ] j k ϵ 0 = 0
Es suficiente para alcanzar por encima de eq. (014) de la ecuación de Euler-Lagrange (002) con respecto η k = A k , k = 1 , 2 , 3 :

(015) t ( L A ˙ k ) + [ L ( A k ) ] L A k = 0

Ahora

(016a) L A ˙ k = A ˙ k ( ϕ A ˙ + 1 2 A ˙ 2 ) = ϕ X k + A k t

(016b) L A k = A k ( j A ϵ 0 ) = j k ϵ 0
y
(016c) L ( A k ) = ( A k ) ( 1 2 C 2 k = 1 k = 3 [ A X k A k A k 2 ] ) = C 2 ( A X k A k )
La última ecuación en (016c) es válida debido a la identidad (Id-02) demostrada en 2. Sección de Identidades :
(Id-02) ( | | × A | | 2 ) ( A k ) = ( A k ) ( k = 1 k = 3 [ A X k A k A k 2 ] ) = 2 ( A k A X k )
Utilizando las expresiones de las ecuaciones (016), la ecuación de Euler-Lagrange (015) da (014) y, por lo tanto, la ecuación de Maxwell (001b).

2. Sección de Identidades

Si A = ( A 1 , A 2 , A 3 ) es una función vectorial de las coordenadas cartesianas ( X 1 , X 2 , X 3 ) después

(Id-01) × A 2 k = 1 k = 3 [ A k 2 A X k A k ]
y
(Id-02) ( | | × A | | 2 ) ( A k ) = ( A k ) ( k = 1 k = 3 [ A X k A k A k 2 ] ) = 2 ( A k A X k )
donde la derivada funcional del lado izquierdo se define como
(Id-03) ( | | × A | | 2 ) ( A k ) [ ( | | × A | | 2 ) ( A k X 1 ) , ( | | × A | | 2 ) ( A k X 2 ) , ( | | × A | | 2 ) ( A k X 3 ) ]
Prueba de ecuación (Id-01) :
| | × A | | 2 = ( A 3 X 2 A 2 X 3 ) 2 + ( A 1 X 3 A 3 X 1 ) 2 + ( A 2 X 1 A 1 X 2 ) 2 = [ ( A 1 X 2 ) 2 + ( A 1 X 3 ) 2 ] + [ ( A 2 X 1 ) 2 + ( A 2 X 3 ) 2 ] + [ ( A 3 X 1 ) 2 + ( A 3 X 2 ) 2 ] 2 [ A 1 X 2 A 2 X 1 + A 2 X 3 A 3 X 2 + A 3 X 1 A 1 X 3 ] = [ ( A 1 X 1 ) 2 + ( A 1 X 2 ) 2 + ( A 1 X 3 ) 2 ] + [ ( A 2 X 1 ) 2 + ( A 2 X 2 ) 2 + ( A 2 X 3 ) 2 ] + [ ( A 3 X 1 ) 2 + ( A 3 X 2 ) 2 + ( A 3 X 3 ) 2 ] [ ( A 1 X 1 ) 2 + ( A 2 X 2 ) 2 + ( A 3 X 3 ) 2 ] 2 [ A 1 X 2 A 2 X 1 + A 2 X 3 A 3 X 2 + A 3 X 1 A 1 X 3 ] = A 1 2 + A 2 2 + A 3 2 ( A 1 X 1 A 1 X 1 + A 2 X 1 A 1 X 2 + A 3 X 1 A 1 X 3 ) ( A 1 X 2 A 2 X 1 + A 2 X 2 A 2 X 2 + A 3 X 2 A 2 X 3 ) ( A 1 X 3 A 3 X 1 + A 2 X 3 A 3 X 2 + A 3 X 3 A 3 X 3 ) = A 1 2 + A 2 2 + A 3 2 A X 1 A 1 A X 2 A 2 A X 3 A 3 = k = 1 k = 3 [ A k 2 A X k A k ]
Prueba de ecuación (Id-02) : De ecuación
| | × A | | 2 = ( A 3 X 2 A 2 X 3 ) 2 + ( A 1 X 3 A 3 X 1 ) 2 + ( A 2 X 1 A 1 X 2 ) 2 = [ ( A 1 X 2 ) 2 + ( A 1 X 3 ) 2 ] + [ ( A 2 X 1 ) 2 + ( A 2 X 3 ) 2 ] + [ ( A 3 X 1 ) 2 + ( A 3 X 2 ) 2 ] 2 [ A 1 X 2 A 2 X 1 + A 2 X 3 A 3 X 2 + A 3 X 1 A 1 X 3 ]
tenemos
( | | × A | | 2 ) ( A 1 X 1 ) = 0 = 2 ( A 1 X 1 A 1 X 1 ) ( | | × A | | 2 ) ( A 1 X 2 ) = 2 ( A 1 X 2 A 2 X 1 ) ( | | × A | | 2 ) ( A 1 X 3 ) = 2 ( A 1 X 3 A 3 X 1 )
Asi que
( | | × A | | 2 ) ( A 1 ) = 2 ( A 1 A X 1 )
demostración de la ecuación (Id-02) para k = 1 y de manera similar para los otros dos componentes k = 2 , 3 .

Un método es variar la acción de Maxwell (establecer j m = 0 si quieres, para el caso sin fuente)

S = d 4 X L = d 4 X ( 1 4 F m v F m v + j m A m ) .
Primera nota que
d ( F m v F m v ) = 2 F m v d F m v = 2 F m v ( m d A v v d A m ) = 4 F m v m d A v = 4 [ m ( F m v d A v ) m F m v d A v ] ,
donde hemos usado el hecho de que F es antisimétrico.

Note también que el m ( F m v d A v ) desaparecerá al convertirlo en una integral de superficie, utilizando el argumento estándar de que d A m desaparece en el límite de integración.

Usando lo anterior, la variación de la acción es

d S = d 4 X   d A v ( m F m v + j v ) ,
que, desde d A v es arbitrario, produce el resultado deseado
m F m v = j v .

en lugar de asumir d A m desapareciendo en el límite se puede suponer F m v en el límite. ¿Es eso incorrecto? Vea una pregunta relacionada aquí physics.stackexchange.com/questions/438277/… @EricAngle